Which of the equations or inequalities below are true?O A. 7-26O B. 7-224O C. 7-23 5O D. 7 - 2 = -5

Which Of The Equations Or Inequalities Below Are True?O A. 7-26O B. 7-224O C. 7-23 5O D. 7 - 2 = -5

Answers

Answer 1

Given,

The equation orr inequalities are,

[tex]\begin{gathered} A)7-2\ge6 \\ B)7-2\ne4 \\ C)7-2\leq5 \\ D)7-2=-5 \end{gathered}[/tex]

A) In the expression,

7 - 2 = 5

Hence, 7-2 =6 is incorrect.

B) In the expression,

7 - 2 = 5,

Hence,

[tex]7-2\ne4[/tex]

Option B is correct.

C) In the expression,

7 - 2 = 5

7 - 2 < 5 is incorrect

Hence, 7 - 2 =< 5 is incorrect.

D)In the expression,

7 - 2 = 5

Hence, 7 - 2 =< -5 is incorrect.

Hence, option B is correct.


Related Questions

Given a and b are the first-quadrant angles, sin a=5/13, and cos b=3/5, evaluate sin(a+b)1) -33/652) 33/653) 63/65

Answers

We know that angles a and b are in the first quadrant. We also know this values:

[tex]\begin{gathered} \sin a=\frac{5}{13} \\ \cos b=\frac{3}{5} \end{gathered}[/tex]

We have to find sin(a+b).

We can use the following identity:

[tex]\sin (a+b)=\sin a\cdot\cos b+\cos a\cdot\sin b[/tex]

For the second term, we can replace the factors with another identity:

[tex]\sin (a+b)=\sin a\cdot\cos b+\sqrt[]{1-\sin^2a}\cdot\sqrt[]{1-\cos^2b}[/tex]

Now we know all the terms from the right side of the equation and we can calculate:

[tex]\begin{gathered} \sin (a+b)=\sin a\cdot\cos b+\sqrt[]{1-\sin^2a}\cdot\sqrt[]{1-\cos^2b} \\ \sin (a+b)=\frac{5}{13}\cdot\frac{3}{5}+\sqrt[]{1-(\frac{5}{13})^2}\cdot\sqrt[]{1-(\frac{3}{5})^2} \\ \sin (a+b)=\frac{15}{65}+\sqrt[]{1-\frac{25}{169}}\cdot\sqrt[]{1-\frac{9}{25}} \\ \sin (a+b)=\frac{15}{65}+\sqrt[]{\frac{169-25}{169}}\cdot\sqrt[]{\frac{25-9}{25}} \\ \sin (a+b)=\frac{15}{65}+\sqrt[]{\frac{144}{169}}\cdot\sqrt[]{\frac{16}{25}} \\ \sin (a+b)=\frac{15}{65}+\frac{12}{13}\cdot\frac{4}{5} \\ \sin (a+b)=\frac{15}{65}+\frac{48}{65} \\ \sin (a+b)=\frac{63}{65} \end{gathered}[/tex]

Answer: sin(a+b) = 63/65

Use the sequence below to complete each task. 34, 25, 16, 7, ... a. Identify the common difference (a). b. Write an equation to represent the sequence. c. Find the 20th term (azo)

Answers

Problem

Solution

We have the following sequence of terms 34,25,16,7,....

Part a

The common difference for this case would be:

25-34= -9

16-25=-9

7-16= -9

Then the answer for part a would be -9

Part b

We want to write the following form:

an = a1 + (n-1) d

For this case d=-9, a1= 34

And then we can write the genral expression like this:

an = 34 + (n-1 ) (-9)

With n = 1,2,3,4....

Part c

In order to find the 20 th term we can replace n =20 and we got:

a20= 34 + (20-1) (-9) = 34-171= -137

I need help with a 8th-grade math assignment:China has a population of approximately 1,382,323,332 people. The United States has a population of about 324,118,787 people.Part AUsing scientific notation, give the approximation for each population. Round the first factor to the nearest tenth.China: United States: Part BAbout how many more people live in China than in the United States? Express your answer using scientific notation. Round the first factor to the nearest hundredth.

Answers

We have the next given information:

China has a population of approximately 1,382,323,332 people.

The United States has a population of about 324,118,787 people.

a) We need to use scientific notation which is given by the next form:

[tex]ax10^n[/tex]

Where a is the coefficient rounded to the nearest tenth.

and a is the terms moved to the decimal point.

For China:

1382323332.0, we moved the decimal point nine spaces to the right.

then

1.382323332

Expressed in scientific notation and rounded to the nearest tenth:

[tex]1.4x10^9[/tex]

For the United States:

324,118,787 with the decimal point 324,118,787.0

then

324118787.0

Move the decimal point 8 spaces to the right, then:

3.24118787

Expressed in scientific notation and rounded to the nearest tenth:

[tex]3.2x10^8[/tex]

Part b:

To find how many more people live in China than in the United States, we need to subtract between China and the United States:

Then:

1,382,323,332 - 324,118,787 = 1,058,204,545

Now

1,058,204,545 equal to 1058204545.0

We need to move the decimal point 9 spaces:

1.058204545

Expressed as scientific notation and rounded to the nearest hundredth:

[tex]1.06x10^9[/tex]

nction.
f(x) = -x² + 3x + 11
Find f(-1)

Answers

Answer:

f(-1) = 7

Step-by-step explanation:

Hello!

You can evaluate for f(-1) by substituting -1 for x in the equation.

Evaluate f(-1)f(x) = -x² + 3x + 11f(-1) = -(-1)² + 3(-1) + 11f(-1) = -1 -3 + 11f(-1) = -4 + 11f(-1) = 7

f(-1) is 7.

f(-1) = -(-1)^2 + 3(-1) + 11
f(-1) = 2 + -3 + 11
f(-1) = 10

Answer:

f(-1) = 7

Step-by-step explanation:

Hello!

You can evaluate for f(-1) by substituting -1 for x in the equation.

Evaluate f(-1)f(x) = -x² + 3x + 11f(-1) = -(-1)² + 3(-1) + 11f(-1) = -1 -3 + 11f(-1) = -4 + 11f(-1) = 7

f(-1) is 7.

f(-1) = -(-1)^2 + 3(-1) + 11
f(-1) = 2 + -3 + 11
f(-1) = 10

could you help me no other tutor will help and its heartbreaking so please try your hardest

Answers

The triangle has sides

a=8

b=14

c=19

You need to determine the measure of x

To determine the value of x you have to use the Law of Cosines that states that:

[tex]a^2+b^2-ab\cos \theta=c^2[/tex]

Where a, b, and c are the sides of the triangle, and theta represents the angle we are looking for.

So first step is to replace the formula with the given data and solve the exponents

[tex]\begin{gathered} 8^2+14^2-8\cdot14\cos thetha=19^2 \\ 64+196-112\cos \theta=361 \\ 260-112\cos \theta=361 \end{gathered}[/tex]

Next solve for the cosine of theta:

[tex]\begin{gathered} -112\cos \theta=361-260 \\ -112\cos \theta=101 \\ \cos \theta=\frac{101}{-112} \\ \cos \theta=-\frac{101}{112} \end{gathered}[/tex]

And calculate the inverse cosine to determine the measure of the angle

[tex]\begin{gathered} \theta=\cos ^{-1}(-\frac{101}{112}) \\ \theta=154.39 \end{gathered}[/tex]

What is the constant of proportionality of x 0 4 8 12 y 0 3 6 9

Answers

Answer:

3/4

Step-by-step explanation:

As y is changing by 3, x is changing by 4

Eliminate the y in the following system of equations. What is the result when you add the two equations? [tex]x + y = 8 \\ 5x - 3y = 24[/tex]A: 6x = 32B: 8x = 32 C: x = 0D: 8x = 48

Answers

EXPLANATION

x + y = 8 ----------------------------------------(1)

5x - 3y = 24 ------------------------------------------(2)

If we are to eliminate y in the equations, we first need to multiply through equation (1) by 3.

3x + 3y = 24 ----------------------------------------(3)

Add equation (2) and equation (3).

If we add equation(1) and equation(3) together, -3y will cancel-out 3y.

(5x + 3x) = (24 + 24)

8x = 48

Therefore, the correct option is D. 8x = 48

The pie chart below shows the percentage of total revenue that a publisher receives from publications.A) which category accounts for approximately 1/5 of publishers total revenue type in which one (textbooks, cookbooks, magazines, paperbacks,poetry, nonfiction B)Approximately percentage total revenue from paperbacks poetry combined C) if revenue from nonfiction 10% total revenue approximately percentage total comes from poetry

Answers

a) To determine which category represents approximately one-fifth of the publisher's revenue, you can compare the given pie chart with a pie chart divided into 5 parts.

The category that is approximately one-fifth of the revenue, that is, 20% of the total revenue, is textbooks.

To solve parts b and c you can use the following pie chart to compare with the given chart for the publisher's revenue:

b) Looking at the pie charts, the categories "paperback" and "poetry" are approximately 20% of the total revenue.

c) The category "Nonfiction" is 10% of the total revenue, the category "poetry" is approximately half of the size of the category "nonfiction" so you can say that it corresponds to 5% of the total revenue.

AlgebraGraphing Linear EquationsHow Did The Poet Write To His Love?Graph each set of equations on its given coordinate plane:Graph: x-3y-mx+BGraph: x = -2X-2y=-2xGraph: y=-3x - 4Graph: x=-4Graph: y =Graph: x=-5Graph: x = -5y=x-sy = 3x - 4y = 4y1yosy-X + 3--

Answers

[tex]\begin{gathered} y=-3x-4 \\ y=3x-4 \end{gathered}[/tex]

This lines are 2 lines that cross over the same intercept in the y-axis.

they both have the y-intercept: -4

they also have the same slope but one on them is negative which makes the line cross each other at the y-intercept.

The graph should be:

Find the common difference and the recursive formula. 22,19,16,13

Answers

[tex]22,19,16,13[/tex]

The common difference between each term is -3.

19 - 22 = -3

16 - 19 = -3

13 - 16 = -3

The recursive formula of an arithmetic sequence follows the pattern below:

[tex]a_n=a_{n-1}+d,n\ge2[/tex]

where d = common difference and number of terms "n" must be more than or equal to two.

To be able to get the recursive formula, we will plug in the common difference assuming that first term a₁ = 22. Therefore, the recursive formula is:

[tex]a_n=a_{n-1}-3,for\text{ n}\ge2[/tex]

Please help! I have 3 questions that I'm not sure what to do about :( if you could answer them that would be great

Answers

The resultant matrix for the given expressions are as follows -

[A]  → 2B - C = [tex]\begin{pmatrix}7.5&2.4\\ -16.4&-8.8\end{pmatrix}[/tex]

[B] → C + 2A = [tex]\begin{pmatrix}18.5&-8.4\\ 16.4&6.8\end{pmatrix}[/tex]

[C] → 2B - 10C + A = [tex]\begin{pmatrix}12&-36\\ -31&25\end{pmatrix}[/tex]

What is a Matrix?

Matrix is a rectangular array or table of numbers, symbols, or expressions, arranged in rows and columns. A matrix has [n] rows and [m] columns. The order of a matrix is given by - [m] x [n].

Given are the three different matrices as -

A = [tex]\left[\begin{array}{ccc}9&-6\\7&5\\\end{array}\right][/tex]

B = [tex]\left[\begin{array}{ccc}4&3\\-7&-6\\\end{array}\right][/tex]

C = [tex]\left[\begin{array}{ccc}0.5&3.6\\2.4&-3.2\\\end{array}\right][/tex]

We can solve the given expressions as follows.

[A]  → 2B - C = [tex]\begin{pmatrix}7.5&2.4\\ -16.4&-8.8\end{pmatrix}[/tex]

[B] → C + 2A = [tex]\begin{pmatrix}18.5&-8.4\\ 16.4&6.8\end{pmatrix}[/tex]

[C] → 2B - 10C + A = [tex]\begin{pmatrix}12&-36\\ -31&25\end{pmatrix}[/tex]

Therefore, the resultant matrix for the given expressions are as follows -

[A]  → 2B - C = [tex]\begin{pmatrix}7.5&2.4\\ -16.4&-8.8\end{pmatrix}[/tex]

[B] → C + 2A = [tex]\begin{pmatrix}18.5&-8.4\\ 16.4&6.8\end{pmatrix}[/tex]

[C] → 2B - 10C + A = [tex]\begin{pmatrix}12&-36\\ -31&25\end{pmatrix}[/tex]

To solve more questions on Matrix, visit the link below-

https://brainly.com/question/12579158

#SPJ1

Finding a specify term of a geometric sequence given the common ratio and first term

Answers

Explanation

A geometric sequence is defined as:

[tex]\begin{gathered} a_1=a*r^0=a*r^{1-1}, \\ a_2=a*r^1=a*r^{2-1}, \\ a_3=a*r^2=a*r^{3-1}, \\ ... \\ a_7=a*r^6=a*r^{7-1}, \\ ... \end{gathered}[/tex]

Where r ≠ 0 is the common ratio and a ≠ 0 is the first term of the sequence.

From the statement, we know that r = 2/3 and the first term is a = 5.

Replacing these numbers in the expression of the 7th term, we get:

[tex]a_7=5*(\frac{2}{3})^6=5*\frac{64}{729}=\frac{320}{729}.[/tex]Answer

320/729

solve p(x+q)^4=r for x

Answers

Given the following equation:

[tex]p\mleft(x+q\mright)^4=r[/tex]

You can solve for the variable "x" as following:

1. You need to apply the Division property of equality by dividing both sides of the equation by "p":

[tex]\begin{gathered} \frac{p\mleft(x+q\mright)^4}{p}=\frac{r}{p} \\ \\ \mleft(x+q\mright)^4=\frac{r}{p} \end{gathered}[/tex]

2. Remember that:

[tex]\sqrt[n]{a^n}=a[/tex]

Then:

[tex]\begin{gathered} \sqrt[4]{(x+q)^4}=\sqrt[4]{\frac{r}{p}} \\ \\ x+q=\sqrt[4]{\frac{r}{p}} \end{gathered}[/tex]

3. Now you have to apply the Subtraction property of equality by subtracting "q" from both sides of the equation:

[tex]\begin{gathered} x+q-(q)=\sqrt[4]{\frac{r}{p}}-(q) \\ \\ x=\sqrt[4]{\frac{r}{p}}-q \end{gathered}[/tex]

The answer is:

[tex]x=\sqrt[4]{\frac{r}{p}}-q[/tex]

Hello! Need some help on part c. The rubric, question, and formulas are linked. Thanks!

Answers

Explanation:

The rate of increase yearly is

[tex]\begin{gathered} r=69\% \\ r=\frac{69}{100}=0.69 \end{gathered}[/tex]

The number of lionfish in the first year is given beow as

[tex]N_0=9000[/tex]

Part A:

To figure out the explicit formula of the number of fish after n years will be represented using the formula below

[tex]P(n)=N_0(1+r)^n[/tex]

By substituting the formula, we will have

[tex]\begin{gathered} P(n)=N_{0}(1+r)^{n} \\ P(n)=9000(1+0.69)^n \\ P(n)=9000(1.69)^n \end{gathered}[/tex]

Hence,

The final answer is

[tex]f(n)=9,000(1.69)^n[/tex]

Part B:

to figure out the amoutn of lionfish after 6 years, we wwill substitute the value of n=6

[tex]\begin{gathered} P(n)=9,000(1.69)^{n} \\ f(6)=9000(1.69)^6 \\ f(6)=209,683 \end{gathered}[/tex]

Hence,

The final answer is

[tex]\Rightarrow209,683[/tex]

Part C:

To figure out the recursive equation of f(n), we will use the formula below

From the question the common difference is

[tex]d=-1400[/tex]

Hence,

The recursive formula will be

[tex]f(n)=f_{n-1}-1400,f_0=9000[/tex]

[tex]((1.25 \times {10}^{ - 15} ) \times (4.15 \times {10}^{25} )) \div ((2.75 \times {10}^{ - 9}) \times (3.4299 \times {10}^{8} ))[/tex]solve. final answer in scientific notation

Answers

[tex]\begin{gathered} \frac{1.25x10^{-15}x4.15x10^{25}}{2.75x10^{-9}x3.4299x10^8} \\ \frac{5.1875x10^{10}}{0.9432} \\ 5.4999x10^{10} \end{gathered}[/tex]

done

[tex]\text{result = 5.4999 x 10}^{10}[/tex]

[tex]Inscientificnotation=5.4999x10^0[/tex]

which question is best modeled with a division expression?
how many 3/8's are in 48
what is 3/8 of 48
what is the total of 3/8 and 48
how much more than 3/8 is 48

Answers

The question that can be best modeled as a division expression is how many 3/8's are in 48.

What is division?

Division is the process of dividing a number into equal parts using another number. The sign used to denote division is ÷. Division is one of the basic mathematical operations. The other mathematical operations are addition, multiplication and division.

In order to determine the 3/8's in 48, divide 48 by 3/8.

In order to determine 3/8 of 48, multiply 48 by 3/8.

In order to determine the total of 3/8 and 48, add the numbers together.

In order to determine how much more than 3/8 is 48, subtract 3/8 from 48.

To learn more about division, please check: https://brainly.com/question/13281206

#SPJ1

A straight line is 180 degrees. Find the value of X.

Answers

Given a straight line angle = 180

So, the angles (9x-100) and (40-x) are supplementary angles

So,

[tex](9x-100)+(40-x)=180[/tex]

Solve for x:

[tex]\begin{gathered} (9x-x)+(40-100)=180 \\ 8x-60=180 \\ 8x=180+60 \\ 8x=240 \\ x=\frac{240}{8}=30 \end{gathered}[/tex]

So, the answer will be x = 30

How many different 3 digit combinations can there be for a combination lock that has a six digit wheel?

Answers

The number of 3 digit combinations possible for the six digit wheel combination lock is; 216 combinations.

Combinations and selections.

It follows from the task content that the number of possible 3 digit combinations for the six digit wheel as required is to be determined.

The number of possible selections in a given sample space is defined by the combination which defines the situation.

On this note, each digit from the 3 digit combinations could be any of the six digits on the wheel.

Therefore, the number of possible combinations is; 6 × 6 × 6 = 216 combinations.

Ultimately, the number of possible combinations is; 216 combinations.

Read more on combinations;

https://brainly.com/question/28065038

#SPJ1

Given the figure below, determine the angle that is a same side interior angle with respect to

Answers

We remember that two interior angles are those inside the are of the lines, Thus, the angles in the area:

Are interior. Now, we identify two sides, the right side, and the left side, which have been separated by the transversal line.

Thus, the angle that is is the same side as ∡3, and also that is interior is ∡5.

Hello! By the way when answering the question just don’t mind my work shown or my answer I know for a fact I am wrong.

Answers

We have to calculate the height of the stack of hay bales.

We can start by calculating the volume as the number of bales times the volume of one hay:

[tex]\begin{gathered} V=n*V_0=8*(10+\frac{2}{3}) \\ V=8*10+8*\frac{2}{3} \\ V=80+\frac{16}{3} \\ V=80+\frac{15}{3}+\frac{1}{3} \\ V=80+5+\frac{1}{3} \\ V=85+\frac{1}{3} \end{gathered}[/tex]

Now, we know that this volume will be the area of the base times the height.

The area of the base can be calculated as the product of the length and the width:

[tex]\begin{gathered} A_b=L*W \\ A_b=4*(1+\frac{1}{3}) \\ A_b=4+\frac{4}{3} \\ A_b=\frac{4*3+4}{3} \\ A_b=\frac{12+4}{3} \\ A_b=\frac{16}{3} \end{gathered}[/tex]

We then can calculate the height as the volume divided by the base area:

[tex]\begin{gathered} h=\frac{V}{A} \\ h=\frac{85+\frac{1}{3}}{\frac{16}{3}} \\ h=\frac{85*3+1}{3}*\frac{3}{16} \\ h=\frac{256}{3}*\frac{3}{16} \\ h=\frac{256}{16} \\ h=16 \end{gathered}[/tex]

Answer the height is 16 feet.

I need help pls 1. Is this graph sine or cosine 2. What’s the amplitude of graph 3. What’s the equation of the midline 4. Whats the period of the function Whats the equation of the function Whats the domain and range?

Answers

As per given by the question,

There are given that a graph.

Now,

1. The given graph is cosine graph.

2. The aplitute of the given graph is,

From the graph, it is lie between -2 to 2.

So,

The amplitude of the given graph is 2.

Now,

3. The equation of the midline is,

[tex]y=-2[/tex]

Now,

4.The period of the fumction is,

[tex]P=\frac{2\pi}{3}[/tex]

Now,

The equation of the function.

First the general form of cosine graph function is,

[tex]y=A\cos (bx+c)+d[/tex]

Then,

[tex]y=2\cos (3x+c)+d[/tex]

Now,

[tex]y=2\cos (3x-1)+3[/tex]

Where, D is vertical shift.

Hence, the equation of the function is,

[tex]y=2\cos (3x-1)+3[/tex]

WhichIs 9.56556555... a rational or irrationalnumber? Highlight the correct answer below.181a)Whicha) Rational numberb) Irrational numberthat ap

Answers

Answer

Option B is correct.

9.56556555... is an irrational number.

Step-by-step Explanation

Rational numbers are numbers that can be expressed as a clear fraction consisting of the numerator and the denominator both being integers.

The decimal form or decimal expansion of a rational number terminates after a particular/finite number of digits (e.g., 0.25, 0.762 etc.) or begins to repeat/recur the same sequence over and over again (e.g., 0.333..., 0.267267... etc)

Anything other than these two rules, the number is regarded as an irrational number.

The number given is 9.56556555...

The dots indicate thst the numbers after the decimal point conbtinue till eternity.

Observing the numbers after the decimal point for the given number, one can see that 565 repeats once and then the number after the second 565 is 55, indicating that the 565 doesn't recur till infinity.

Since the numbers after the decimal point doesnt contain a finite number of digits and the numbers don't recur till infinity, we can conclude that 9.56556555... isn't a rational number.

Hope this Helps!!!

hi can you see if I did this estimate right?

Answers

Mr Manet need 5 guitars for his 4 grandsons and 1 granddaughter.

Each guitar costs $88,

So the total cost of guitars is $88 x 5 = $440

The best estimate among the choices is Choice B. $450

The estimate should always be higher than the actual cost.

URGENT!! ILL GIVE
BRAINLIEST! AND 100 POINTS

Answers

Answer:

perimeter: add p + m + n

area: use (p × m)/2

find missing side: use p^2 + m^2 = n^2

A bag contains 5 red and 3 blue marbles. Two marbles are drawn simultaneously from the bag. DETERMIN the probability that at least one is red.

Answers

total number of balls = 5 + 3 = 8

The possibilities are:

RR (two red) and RB (one red and one blue)

RR and RB are mutually exclusive

P(RR) =

How many different arrangements of letters can be formed if the letter must be repeats of letters are allowed?

Answers

Solution:

There is 2 possibility for the first letter from the left.

Then there are 2 possibilities for the second letter. Then and so on till the 5-th letter.

In this way, you will get

[tex]2^{5\text{ }}=32[/tex]

Each is unique, and each is achievable in this way.

There are no other arrangements. So that, we can conclude that the correct answer is:

[tex]32[/tex]

Help please!!!!!!!!!!!!!!!!!!!!!!!!!Which is the best buy?a. $18.09 for 9 bottles of juiceb. $22.33 for 11 bottles of juice

Answers

Answer:

The correct option is option a.

$18.08 for 9 bottles is the better buy.

Explanation:

The best buy is the item with the lesser price tag.

Let us chech which of the given items has the lesser price tag.

a. $18.08 for 9 bottles of juice

Let us find how much one bottle costs.

1 bottle = 18.08/9

= 2.01

1 bottle of juice costs $2.01 approximately

b. $22.23 for 11 bottles of juice

1 bottle = 22.23/11

= 2.03

1 bottle of juice costs $2.03 approximately.

Comparing these prices per bottle of juice, we realise that the one with $18.08 for 9 bottles is the better buy.

Leo is constructing a tangent line from point Q to circle P. What is his next step? Mark the point of intersection of circle P and segment PQ. Construct arcs from point P that are greater than half the length of segment PQ. Construct a circle from point Q with the radius PQ. Plot a new point R and create and line perpendicular to segment PQ from point R

Answers

Explanation

The next step to constructing a tangent line from Q to circle P is to construct the perpendicular bisector of the segment PQ.

For this, Leo can construct arcs from point P and from point Q that are greater than half the length of segment PQ.

Answer

The next step is to construct arcs from point P that are greater than half the length of segment PQ.

Solve for x.2x + 3 ≤ x + 5

Answers

Answer:

x ≤ 2

Explanation:

Given the inequality:

[tex]2x+3\le x+5[/tex]

First, we subtract x from both sides.

[tex]\begin{gathered} 2x-x+3\le x-x+5 \\ x+3\le5 \end{gathered}[/tex]

Next, we subtract 3 from both sides.

[tex]\begin{gathered} x+3-3\leqslant5-3 \\ x\leqslant2 \end{gathered}[/tex]

Which of the following tables represents a function?

Answers

Answer:

Table A represents a function

Step-by-step explanation:

Table A represents function because it is the only table that doesn't repeat an output or input number.

Other Questions
A training field is formed by joining a rectangle and two semicircles, as shown below. The rectangle is 81 m long and 60 m wide. Find the area of the training field. Use the value 3.14 for n, and do not round your answer. Be sure to include the correct unit in your answer. Find the volume of a pyramid with a square base, where the side length of the base is11 in and the height of the pyramid is 15.1 in. Round your answer to the nearesttenth An architect plans to buy 5 stone spheres and 3 stone cylinders. For the same amount, he can buy 2 stone spheres and 7 stone cylinders. If one stone cylinder costs $32.73 how much doeseach stone sphere cost? Use the following function rule to find (2). f(x) = (-5 - 2x)? ? f(2) = | Submit if 5 is added eighteen times to a number the result is 174 what is the number The table lists recommended amounts of food to order for 19 party guests. Nathan and Sydney are hosting a graduation party for 40 guests. They know there will also be guests stopping by who may have come from other parties. For ordering purposes, they will count each of these "drop-in" guests as half a guest. How much of each food item should Nathan and Sydney order for a graduation party with 15 drop-in guests?table: fried chicken =28 meats =4 1/3 lasangnia = 10 3/4 Find the exact area of a circle with a diameter of 12 in., expressed in terms of n.361 square inches61 square inches14471 square inches12 square inches241 square inches< Previous U.S. Customary unit conversion with mixed number values: One...Charlie bought 5 feet of fabric. How much is this in yards?Write your answer as a whole number or a mixed number in simplest form. Include the correct unit in your answer.0OlaDOinftydx 5?Please help A new born child receives a $8,000 gift toward a college education from her grandparents. How much will the $8,000 be worth in 17 years if it is invested at 72% compounded quarterly?It will be worth $(Round to the nearest cent) 7. You are single and claim 1 allowance. You presently earn $319 per week.Starting next week you will receive a 5 percent increase in pay and will earn$335.00. How much more will you have withheld from your weekly pay for federal income tax? 1. How much less is the area of a rectangular field 60 by 20 meters than that of a square field with the same perimeter? May I please get help with this. For I have tried many times but still could not get the rotation correct For the following set of data, find the number of data within 2 population standard deviations of the mean.28, 65, 114, 74, 68, 75, 70, 69, 64 . Rellena los espacios en blanco con la forma correcta de SER en el presente. the sum of billiard balls was arranged in an equilateral triangle and 7 balls were extra. Then the same set of billiard balls was arranged into a triangle where each side has one more ball than in the first arrangement but now the new arrangement cannot be completed because there is a shortage of three balls. How many balls are in the set? Which cosine function has maximum of 2, a minimum of 2, and a period of 2pi/3 ? O EQUATIONS AND INEQUALITIESSolving a word problem using a quadratic equation with rationa... On a particular college campus, 22% of the students belong to a fraternity or sorority. If 56 college students are randomly chosen:a. What is the probability that 16 are members of a fraternity or sorority?Round to at least three decimal places.Incorrectb. What is the mean of this distribution? Round to at least one decimal.Incorrectc. What is the standard deviation of this distribution? Round to at least one decimal (7, 8) and (-1, 0)find the distance between the two points? the 30-year, 5.5 percent bonds issued by modern kitchens pay interest semiannually, mature in four years, and have a $1,000 face value. currently, the bonds sell for $1,020.66. what is the yield to maturity?